Answer (A) is correct . The factor to use is 2.5, which is found at a little under 10% on the 3-year line of an annuity table.
Answer (B) is incorrect because Discounting the cash inflows at 12% would not produce a NPV of zero. Answer (C) is incorrect because Discounting the cash inflows at 22% would not produce a NPV of zero. Answer (D) is incorrect because Discounting the cash inflows at 27% would not produce a NPV of zero.
|